Limite d'une suite

Bonjour !

Petit défi

Sauriez vous prouver que si je pose $a = \frac12(1+ i\sqrt7)$ et $u_n = |Re(a^n)|$ alors $u_n \to \infty$

Si vous aviez des idées simples...

Réponses

  • On a un théorème : si pour tout $n$, $u_n=a_n+ib_n$ où $a_n$ et $b_n$ sont réels, alors $|u|$ tend vers l’infini si et seulement si $|a|$ et $|b|$ tendent vers l’infini.

    Édit : un erreur (au moins) à corriger se glisse dans ce théorème
  • Ah ? La suite définie par $u_n=1+n\mathrm{i}$ ($n\in\N$) est un contre-exemple à ce théorème.
  • J’aurais même pris $n\mapsto in$ ou encore $n\mapsto n$.
    Et que dire de $v_n=n $ si $n$ est pair et $v_n= in$ si $n$ est impair ?

    Je laisse l’erreur tiens, il suffit de corriger :-)
  • C'est un peu bourrin mais en développant le binôme de Newton et en utilisant l'inégalité triangulaire inverse on peut montrer ce que tu veux.
  • Regarder dans le blanc des yeux la suite \( v_0 = 1, \; v_1 = \dfrac12, \; \forall n \in \N, \; v_{n+2} = v_{n+1} - 2v_{n} \) ?

    e.v.
    Personne n'a raison contre un enfant qui pleure.


  • J’ai quant à moi fait de l’algèbre linéaire pour avoir le terme général de la partie réelle.
    C’est moche aussi en calculs...
  • L'algèbre linéaire doit pas donner grand chose, tu dois retomber sur l'expression de départ (j'avais essayé aussi)

    Avec $\alpha = arctan(\sqrt7)$
    J'ai essayé d'expliciter les suites $u_n = cos(n.\alpha)$ et $v_n = sin(n.\alpha)$ grâce à une relation de récurrence, algèbre linéaire, valeurs propres... mais ça donne rien, je retombe exactement sur $u_n = \frac12\frac{1}{\sqrt{2^n}}(a^n + \bar{a}^n)$

    J'avais aussi essayé avec $u_n + iv_n = a(u_{n-1} + iv_{n-1})$, même sorte de relation de récurrence et j'obtiens $u_n = \frac12(a^n + \bar{a}^n)$
  • Révérence gardée, je trouve les réponses de Poirot et ev un peu sous-minimales allusives.
  • Bonjour
    En raisonnant par l'absurde, on montre qu'il existe une constante $M>0$ et une infinité d'entiers $p_n$ et $q_n$ tels que
    $\Big|\dfrac{q_n}{p_n}-\sqrt{7}\Big| \leq \dfrac{M}{2 ^n p_n^2} $. Ceci est contradictoire avec le fait que $\sqrt{7}$ est algébrique de degré 2.
     
  • Bonjour,

    > En raisonnant par l'.............

    Attention! En prononçant le mot maudit, tu t'exposes à la réaction du côté obscur de la force :-D.

    Cordialement,

    Rescassol
  • Suite à la remarque renfrognée de Math Coss, je développe un tantinet.
    Car les tantinets aiment à se laisser développer.

    Une fois le blanc des yeux scruté avec circonspection (mais un tableur fait l'affaire aussi bien) on remarque qu'au-delà de \( n = 3 \) on a \( v_n \) du signe de \( (-1)^{n-1} \).
    on étudie alors la suite \( w_3 = 1, \; w_4 = \dfrac12, \; \forall n \geqslant 3, \; w_{n+2} = w_{n+1} + 2w_{n} \), laquelle a le bon goût de tendre vers \( + \infty \).

    e.v.
    Personne n'a raison contre un enfant qui pleure.


  • Hello ev, j'ai l'impression que $v_n$ suit bien ta relation de récurrence. Cependant elle n'est pas du signe de $(-1)^{n-1}$

    -2.5
    0.5
    5.5
    4.5
    -6.5
    -15.5
    -2.5
    28.5
    33.5
    -23.5
    -90.5
    -43.5
    137.5
    224.5
    -50.5
    -499.5


    Et trouver la limite de $|v_n|$ est équivalent à l'énoncé de départ..

    bd, je me penche sur ta solution mais j'ai l'impression qu'il faut montrer que 2^n * partie réelle prend une infinité de valeurs, c'est facile à montrer ça?
  • Je continue à ronchonner : je ne vois pas cette belle alternance de signes sur mon tableur.
    sage: a = (1+i*sqrt(7))/2
    sage: def u(n):
    ....:     return (a^n).real()
    ....: 
    sage: [sign(u(k)) for k in range(20)]
    [1, 1, -1, -1, 1, 1, 1, -1, -1, -1, 1, 1, -1, -1, -1, 1, 1, -1, -1, -1]
    sage: def v(n):
    ....:     L = [1,1/2]
    ....:     for k in range(n):
    ....:         L.append(L[-1]-2*L[-2])
    ....:     return L[n]
    ....: 
    sage: [sign(v(k)) for k in range(20)]
    [1, 1, -1, -1, 1, 1, 1, -1, -1, -1, 1, 1, -1, -1, -1, 1, 1, -1, -1, -1]
    sage: [u(k)-v(k) for k in range(20)]
    [0, 0, 0, 0, 0, 0, 0, 0, 0, 0, 0, 0, 0, 0, 0, 0, 0, 0, 0, 0]
    
  • @Math Coss : j'avais fait une erreur bête, je n'ai pas d'idée.
  • Intéressant, j'ai fini par craquer et ai essayé de voir s'il n'y avait pas quelque chose à ce sujet sur Math SE.
    Et on dirait que je ne suis pas le seul car la question en tête de liste en ce moment est ...

    https://math.stackexchange.com/questions/3839991/limit-of-mathrmrean-where-a-frac121i-sqrt7

    ... posée il y a dix minutes. Je ne comprends pas trop le "downvote". Question intéressante et pas du tout évidente.

    noobey (je suppose que c'est toi qui a posée le question sur Math SE): elle vient vraiment d'un problème de compétition de maths cette question? ça paraît super dur, même pour des génies en herbe.
    Pour l'instant sur Math SE c'est la valse des réponses erronnées qui se rétractent 5 minutes après.
    Après je bloque.
  • On m'a dit que c'était un oral de l'X oui... ce qui semble étrange au vu des outils dont on a visiblement besoin... Après j'en sais pas plus. C'était peut être pas la 1ere question

    Edit : en faisant d'autres recherches j'ai ça :
    https://math.stackexchange.com/questions/961841/proving-that-left-re-left-frac1i-sqrt72-rightn-right-to-infty

    "This appeared in a recent issue of French Revue de la Filière Mathématiques, as it was reportedly asked during an oral exam."
  • Donc N. Elkies montre qu'en fait la suite $u_n=\Re(a_n)$ (sans les valeurs absolues) n'a aucune répétition à partir de $n=4$.

    Sa démonstration est élémentaire mais franchement ingénieuse. On voit mal comment ça pourrait être la preuve suggérée à un oral (pour calculer $u_{n+84}$, difficile de s'en sortir sans un processeur dans la tête).
    Après je bloque.
  • Bonsoir

    Le nombre complexe $a =\frac{1}{2} + i\frac{\sqrt{7}}{2}$
    admet un module $r = \sqrt{2}$ et un argument $t$ tel que $\cos t = \frac{1}{2\sqrt{2}}$
    puisque $\tan t = \sqrt{7}$ avec $\frac{\pi}{4} < t < \frac{\pi}{2}$.

    il vient en conséquence : $Re(a^n) = (\sqrt{2})^n\cos(nt)$ et sa valeur absolue est : $u_n = (\sqrt{2})^n|\cos(nt)|$
    la valeur absolue de $\cos(nt)$ varie entre $0$ et $1$
    E
    n fait la valeur $0$ est exclue car $nt$ ne peut jamais être un multiple impair de $\frac{\pi}{2}$,
    et donc la suite $u(n)$ va diverger vers $+\infty$.

    Cordialement.
  • Le module de $a =\frac{1}{2} + i\frac{\sqrt{7}}{2}$ est $2$, pas $\sqrt{2}$.
    Ensuite, le fait que $\mid\Re(a^n)\mid$ ne soit pas bornée ne siginifie pas que $\mid\Re(a^n)\mid$ tend vers l'infini.
    Après je bloque.
  • JL a écrit:
    et donc la suite u(n) va diverger vers +oo
    Ah bon ? Ça alors ! Mais quand ? Et, plus lancinant, quand aura-t-on une esquisse de preuve ?
  • Bonsoir,

    i.zitoussi, le module de $a$ est bien $\sqrt{2}$, et non $2$.

    Cordialement,

    Rescassol
  • Autant pour moi.

    Cordialement,

    i.zitoussi.
    Après je bloque.
  • Bon j'ai eu la réponse.

    On peut utiliser le théoréme de Skolem Malher Lech.

    Soit (Un) une suite recurrente linéaire. Alors pour n assez grand l'ensemble de ses points d'annulation forment des suites arithmétiques infinies de meme raison r.

    Ensuite 2*Un où Un est la suite recherchée est toujours entier et suit une suite recurrente precisee plus haut par ev.

    Si on montre que pour tout entier k, Un = k a un nombre fini de solutions c'est gagné

    Je pose le corrigé ou vous voulez réfléchir?


    Visiblement on s'est fait arnaquer... c'etait vraiment dur mais ca a été posé d'après la RMS à un oral de concours
  • Bonjour,

    Voici une preuve assez détaillée, adaptable à toutes les suites d'entiers obéissant à une récurrence linéaire, du fait que tout entier n'est atteint qu'un nombre fini de fois par la suite $\left( 2 \Re(a^n)\right) _{n\in \N}.$ De nature exclusivement arithmétique, elle utilise les ressorts de la valuation $p-$ adique , qui sont ceux du théorème de Mahler évoqué par Noobey, et elle fait appel à deux lemmes mentionnés en préambule.

    $\boxed{ \underline{\text{Lemme 1}}\quad\text{ Soient} \:\:g =X^k+ \displaystyle \sum_{j=0}^{k-1} b_jX^j,\:\: h(X) =\sum_{j=0}^m c_jX^j \in \Z[X] , \quad f(X) = g(X) h(X) = \sum _{j=0} ^{m+k} a_jX^j.\\
    \qquad\qquad\quad \text{Alors}, \quad\forall p,n \in \N, \:\: p \:\text{premier},\quad \exists j \in [\![k;k+m]\!], \:\:\: \mathcal V_p(f(n)) \geqslant \mathcal V_p(a_j).}$

    Supposons le contraire: $\:\:\forall j \in [\![k;k+m]\!], \:\:\: \mathcal V_p\big(f(n)\big) < \mathcal V_p(a_j)$.
    En écrivant successivement les coefficients $a_{m+k}, a_{m+k-1} \dots a_k$ de $f(X)$ en fonction de ceux de $g(X)$ et de $h(X)$, on obtient: $\:\:\mathcal V_p\big(f(n)\big) < \mathcal V_p(c_m),\quad \mathcal V_p\big(f(n)\big) < \mathcal V_p(c_{m-1}), \dots\mathcal V_p\big(f(n)\big) < \mathcal V_p(c_0).\quad (\star)$
    Alors: $\: \mathcal V_p\big(f(n)\big) = \mathcal V_p\big(g(n)h(n)\big)\geqslant \mathcal V_p\big(h(n)\big) = \mathcal V_p\big(\sum_{j=0}^m c_jX^j )\big)\overset {(\star)}> \mathcal V_p\big(f(n)\big), \:\:$ ce qui constitue une contradiction $\square$

    $\boxed{\underline {\text {Lemme 2}}\qquad\forall p,n \in \N, \:\: p \:\text{premier}, \:\:\: \mathcal V_p(n!) \leqslant \dfrac n {p-1}.}$
    $\displaystyle \mathcal V_p(n!)= \sum_{k=1}^{+\infty} k \: \text{Card}\left\{ a \in [\![1;n]\!] \mid a\equiv 0 \mod p^k,a \not \equiv 0 \mod p^{k+1}\right \} = \sum_{k=1}^{+\infty} k \left ( \Big\lfloor \dfrac n{p^k}\Big\rfloor -\Big\lfloor \dfrac n{p^{k+1}}\Big\rfloor \right) = \sum_{k=1}^{+\infty} \Big\lfloor \dfrac n{p^k}\Big\rfloor\leqslant \dfrac n{p-1}\: \square$


    Soit $(v_n)_{n\in \N}$ la suite définie par: $ \quad v_0= 2, \:v_1 =1, \:\:\forall n \in \N , \:\:v_{n+2} = v_{n+1} - 2v_n.$
    Alors: $\quad\forall n \in \N,\:\: |v_n| = 2u_n,\:\: v_n \in \Z.\:\: $ Pour démontrer que $\displaystyle \lim_{n\to + \infty} u_n = + \infty,\:\:$ il "suffit" de prouver que: $$ \quad \forall k \in [\![0;7]\!],\:\: \forall p \in \Z,\:\: \{n \in \N \mid v_{k+8n} = p \}\:\text{ est fini.}\qquad(\bigstar)$$
    (cela entraîne en effet que $\displaystyle \lim_{n\to + \infty}|v_{k+8n}| = + \infty,\quad$ puis $\displaystyle \lim_{n\to + \infty}|v_{n}| = + \infty.) $
    Notons: $ \forall n \in \N, \:X_n=\begin{pmatrix} v_{n+1}\\v_n \end{pmatrix},\quad A = \begin{pmatrix}1&-2\\ 1&0 \end{pmatrix},\quad B= \begin{pmatrix} -6&2\\ -1&-5 \end{pmatrix}, \quad \Phi \in \mathcal L\Big (\mathcal M_2(\Z) , \Z\Big):\:\:\begin{pmatrix} a&b\\ c&d \end{pmatrix}\longmapsto c+2d..$
    Alors: $\quad \forall n \in \N,\:\:X_0 =\begin{pmatrix} 2\\1 \end{pmatrix},\quad X_n=A^n X_0, \:\: v_n = \Phi (A^n),\:\: A^8 = \mathrm I_2+3B.\qquad$
    On remarque d'abord que $\forall k \in [\![0;7]\!],\:$ la suite $(v_{k+8n})_n $ n'est pas constante à partir d'un certain rang.
    En effet: $\:\:\:\forall k \in [\![0;7]\!], (v_k,v_{k+8})\neq (0,0) $ et $A^8 - \mathrm I_2$ est inversible.

    Supposons que: $\:(\bigstar)$ soit faux.
    Alors: il existe $ m \in \N$ et une suite $(n_1, n_2 \dots)$ strictement croissante d'entiers tels que: $\:\:\forall k \in \N^*, \:v_m = v_{m+8n_k}.\:\:$
    D'après la remarque précédente: $\:\:\exists \:s, k_0 \in \N^*\: $ tels que $ \:v_{m +8s}\neq v_m, \:\: \:\:n_ {k_0}>s.\quad$ D'autre part:
    $$\forall n \in \N\qquad v_{m +8n} =\Phi \Big(A^m( \mathrm I_2+3B)^n\Big) = v_m + \displaystyle \sum_{i=1} ^n \binom ni 3^i \Phi(A^mB^i).$$
    Notons encore $\Phi(A^mB^i)=t_i \in \Z,\:$ et $\:\quad\forall N\in \N, \:\: f_N (X) =\displaystyle \sum _{i=1}^N 3^i t_i \binom Xi =\sum_{i=1}^N a_i X^i \in \Z[X].$
    Soit $\:k \geqslant k_0\:\:$ et $\:\:N>n_k.\quad \:\:\forall n\leqslant N, \:\:v_{m+8n} -v_m = f_N(n).\:\:\quad $ On a alors: $\forall j \in [\![1;k]\!],\:\: f_N(n_j) =0, \:\:\: f_N(s) \neq 0.$

    Notons enfin: $g_N(X)=\displaystyle \prod _{j=1}^k (X-n_j) =X^k +\sum _{i=0}^{k-1} b_jX^j,\quad f_N(X) =g_N(X) h_N(X), \:\:\: g_N, h_N \in \Z[X].$
    On peut appliquer le lemme $1$ aux polynômes $f_N,g_N,h_N: \:\: \exists j \in [\![k;N\!]], \:\:\: \mathcal V_3(f_N(s)) \geqslant \mathcal V_3(a_j).$
    Tous les coefficients de $\displaystyle \sum _{i=j}^N 3^i t_i \binom Xi $ ont une $3\text{-valuation}$ qui, d'après le lemme $2$, est supérieure à $ \dfrac j2 \:$et $a_j$ est un coefficient de ce polynôme.
    Il s'ensuit que $\:\: \mathcal V_3(f_N(s)) \geqslant \dfrac j2 \geqslant \dfrac k2.\quad$ Ainsi : $\forall k\geqslant k_0,\:\: \mathcal V_3(f_N(s)) \geqslant \dfrac k2$ et cela contredit $f_N(s) \neq 0.$
Connectez-vous ou Inscrivez-vous pour répondre.